3
$\begingroup$

Consider a vector $p =(p_1,...p_n)$, $p_i>0$, $\sum p_i = 1$ and a matrix $M_{ij}$, which is doubly stochastic: $\sum_i M_{ij} = 1, \sum_j M_{ij} = 1, M_{ij} > 0$.

Question 1 Just apply matrix M to a vector $p$ , i.e. $q = Mp$ (i.e. $q_i = \sum_j M_{ij} p_j$) is it true that entropy of new vector $q$ is greater then of original vector $p$ ? I.e.

$$ H(q) = -\sum_i q_i ln(q_i) > -\sum_i p_i ln(p_i) = H(p) $$

Question 2 Is there a simple proof of it ? (It might follow from the Gibb's inequality, but it does not seem obvious for me).

Question 3 What are generalizations, in view of meta-principle "entropy always grows" that might be an example of some more general phenomena ?


Motivation: There is a meta-principle that entropy grows is some natural systems, matrix applied to a vector is probably the most simple system one can consider (Markov chain), so the question above arises. After some thinking one can restrict from all matrices to doubly stochastic, because $M^n v$ tends to a uniform distribution (which has maximal entropy of all) only for doubly stochastic $M$ , so it cannot be true for general $M$.

$\endgroup$
2

4 Answers 4

4
$\begingroup$

This is a particular case of the following general principle. On the one hand, a bistochastic matrix $M$ has the property that for every non-negative vector (say, a probability vector), $Mp\succ p$. On another hand, the order $\succ$ can be defined by $x\succ y$ iff $\sum_if(x_i)\le\sum_if(y_i)$ for every convex function $f$.

Now, use the fact that $f=-H$ is convex.

For proofs, see my book Matrices (Springer-Verlag GTM #216, second edition) at sections 6.5 (Proposition 6.4) and 8.5 (Theorem 8.5).

Edit (at Alexander's request.) Let $\alpha\ne1$ be a positive parameter. The Renyi entropy is $$\frac1{1-\alpha}\log\sum_jp_j^\alpha.$$ The map $p\mapsto Mp$ does increase Renyi entropy. Proof: if $\alpha>1$, the map $t\mapsto t^\alpha$ is convex, and $s\mapsto\frac1{1-\alpha}\log s$ is increasing. If instead $\alpha<1$, the map $t\mapsto t^\alpha$ is concave, and $s\mapsto\frac1{1-\alpha}\log s$ is decreasing.

$\endgroup$
5
  • $\begingroup$ Thank you. That seems to cover Renyi entropy also, is not it? $\endgroup$ Dec 1, 2018 at 19:32
  • $\begingroup$ @AlexanderChervov. Yes, see my edit. $\endgroup$ Dec 2, 2018 at 7:23
  • $\begingroup$ Thank you ! I should buy your book ))) $\endgroup$ Dec 11, 2018 at 19:04
  • $\begingroup$ @AlexanderChervov. It is concise, and stands at the graduate level at every chapter but the two first. Visit also my web page perso.ens-lyon.fr/serre/DPF/exobis.pdf with 469 exercises (up today) which display additional material. $\endgroup$ Dec 11, 2018 at 19:27
  • $\begingroup$ Great collection of exercises ! I sent you a mail may be you add some from that... $\endgroup$ Dec 11, 2018 at 20:04
3
$\begingroup$

The answer to Question 1 is yes as long as $p$ is not the uniform distribution $(\frac{1}{n},\frac{1}{n},\ldots,\frac{1}{n})$.

The proof (Question 2) is quite simple:

Recall from the Birkhoff–von Neumann theorem that every doubly stochastic matrix $M$ is a convex combination of permutation matrices. We can interpret this as saying that there is a distribution $\theta$ on the set of all permutations of $A:=\{1,2,\ldots,n\}$ such that whenever $\mathbf{x}$ is a random variable from $A$ and $\pmb{\pi}$ is a random permutation of $A$ with distribution $\theta$ independent of $\mathbf{x}$, and we set $\mathbf{y}:=\pmb{\pi}(\mathbf{x})$, then we have \begin{align} \mathbb{P}(\mathbf{y}=i\,|\,\mathbf{x}=j) &= M_{i,j} \;. \end{align} Note that if $\mathbf{x}$ is distributed as $p$, then $\mathbf{y}$ is distributed as $q:=Mp$, so $H(\mathbf{x})=H(p)$ and $H(\mathbf{y})=H(q)$.

Now, \begin{align} H(\mathbf{y},\pmb{\pi}) &= H(\mathbf{y}) + H(\pmb{\pi}\,|\,\mathbf{y}) \;, \\ H(\mathbf{y},\pmb{\pi}) &= H(\pmb{\pi}) + \underbrace{H(\mathbf{y}\,|\,\pmb{\pi})}_{H(\mathbf{x})} \;, \end{align} which implies \begin{align} H(\mathbf{y}) &= H(\mathbf{x}) + H(\pmb{\pi}) - H(\pmb{\pi}\,|\,\mathbf{y}) \\ &= H(\mathbf{x}) + I(\mathbf{y};\pmb{\pi}) \end{align} where $I(\mathbf{y};\pmb{\pi})$ is the mutual information between $\mathbf{y}$ and $\pmb{\pi}$. We know that $I(\mathbf{y};\pmb{\pi})\geq 0$ with equality if and only if $\mathbf{y}$ and $\pmb{\pi}$ are independent, which is the case if and only if $\mathbf{x}$ has the uniform distribution. Q.E.D.

For Question 3, suppose that $M$ is merely a positive stochastic matrix (not doubly stochastic). Let $r$ denote the unique stationary distribution of $M$. Then we have a similar ``entropy increase'' principle if we replace entropy with (minus) the Kullback-Leibler divergence relative to $r$. There is a nice discussion on this in the book of Cover and Thomas.

$\endgroup$
2
$\begingroup$

This is a particular case of a well-known monotonicity property of the Kullback-Leibler divergence $D(\cdot\|\cdot)$. Namely, if $\alpha$ and $\beta$ are any two distributions on the same (say, finite) space $X$, and $P$ is a Markov operator on $X$, then $$ D(\alpha\|\beta) \ge D(\alpha P\|\beta P) \;. $$ In your case double stochasticity of $P$ means that its stationary distribution is the uniform distribution $m_X$ on $X$, whereas $$ D(\alpha \| m_X) = \log \text{card} X - H(\alpha) \;. $$

$\endgroup$
1
$\begingroup$

This is true, except that we have equality when $p$ is the uniform distribution, which is the limit distribution of your matrix. (Proof in here, 15.5, for example.)

Whenever $p_i\geq 1/n$, it is not so hard to show that $q_i\leq p_i$, and when $p_i\leq 1/n$, we have $q_i\geq p_i$. From there, it follows that $-\sum_i p_i \ln (q_i) \leq -\sum_i q_i \ln (q_i) $, and this gives the result combined with Gibbs'.

$\endgroup$

Your Answer

By clicking “Post Your Answer”, you agree to our terms of service and acknowledge you have read our privacy policy.

Not the answer you're looking for? Browse other questions tagged or ask your own question.